Solution Final Spring 2021

You might also like

Download as pdf or txt
Download as pdf or txt
You are on page 1of 4

Solution Sample Spring 2021:

Problem 1: (30 points)

𝑥̇ 1 = −𝑥2 + 𝜇𝑥1 (𝑥12 + 𝑥22 )


𝑥̇ 2 = 𝑥1 + 𝜇𝑥2 (𝑥12 + 𝑥22 )
a) Equilibrium points:
𝑥̇ 1 = 0 ⇒ −𝑥2 + 𝜇𝑥1 (𝑥12 + 𝑥22 ) = 0
𝑥̇ 2 = 0 ⇒ 𝑥1 + 𝜇𝑥2 (𝑥12 + 𝑥22 ) = 0
⇒ 𝑥1 = 0, 𝑥2 = 0 , the equilibrium point is (0,0). (4 points)

b) The linearized model is given by:


𝑋̇ = 𝐴𝑋 + 𝐵𝑈
𝜕𝑓1 𝜕𝑓1
| |
𝜕𝑥1 (0,0) 𝜕𝑥2 (0,0)
𝐴=
𝜕𝑓2 𝜕𝑓2
| |
[𝜕𝑥1 (0,0) 𝜕𝑥2 (0,0) ]
𝜇(𝑥12 + 𝑥22 ) + 𝜇𝑥1 (2𝑥1 )|(0,0) −1 + 𝜇𝑥1 (2𝑥2 )|(0,0)
=[ ]
1 + 𝜇𝑥2 (2𝑥1 )|(0,0) 𝜇(𝑥12 + 𝑥22 ) + 𝜇𝑥2 (2𝑥2 )|(0,0)
0 −1
=[ ]
1 0
(4 points)
𝐵=0
𝑋̇ = 𝐴𝑋 (2 points)
c) The eigenvalues are given by:
𝜆 1
𝐷𝑒𝑡(𝜆𝐼 − 𝐴) = 0 ⇒ | | = 𝜆2 + 1 = 0 ⇒ 𝜆 = ±𝑗 (2 points)
−1 𝜆
The equilibrium point is a center. (2 points)
d) We consider the change of variables below:
𝑥1 = 𝑟 cos 𝜃 (1 point)
𝑥2 = 𝑟 sin 𝜃 (1 point)

𝑥̇ 1 = −𝑥2 + 𝜇𝑥1 (𝑥12 + 𝑥22 ) → 𝑟̇ cos 𝜃 − 𝑟𝜃̇ sin 𝜃 = −𝑟 sin 𝜃 + 𝜇 𝑟 cos 𝜃 (𝑟 2 ) (1)

𝑥̇ 2 = 𝑥1 + 𝜇𝑥2 (𝑥12 + 𝑥22 ) → 𝑟̇ sin 𝜃 + 𝑟𝜃̇ cos 𝜃 = 𝑟 cos 𝜃 + 𝜇 𝑟 sin 𝜃 (𝑟 2 ) (2)


Multiplying (1) by cos 𝜃 and (2) by sin 𝜃:
(1) + (2) → 𝑟̇ = 𝜇𝑟 3 (3 points)

Multiplying (1) by sin 𝜃 and (2) by cos 𝜃:

(1) − (2) → 𝑟𝜃̇ = −𝑟 → 𝜃̇ = −1 (3 points)


The system in polar coordinates is given by:
𝑟̇ = 𝜇𝑟 3
{
𝜃̇ = 1
e) If 𝜇 > 0 → unstable focus (2 points)
If 𝜇 < 0 → stable focus (2 points)
If 𝜇 = 0 → center (2 points)

Problem 2: (50 points)


The commutation occurs at 1 − 2𝑥1𝑐 = 0 → 𝑥1𝑐 = 0.5.
First trajectory:
𝑥10 = 0, 𝑥20 = 0
𝑟 − 𝑘𝑀𝑇𝑥1 = 1 − 2𝑥10 = 1 > 0 → 𝜆 = +1
(2 points) (2 points)
𝑥 −𝜆 −1
𝑥1 (𝑡) + 𝑥2 (𝑡) = 𝑥10 + 𝑥20 + 𝜆 ln [𝑥 20
(𝑡)−𝜆
] = 0 + 1 ln [𝑥 ] (2 points)
2 2 −1

𝑥1𝑐 = 0.5 (2 points) , 𝑥2𝑐 =?


−1 −1
0.5 + 𝑥2𝑐 = ln 𝑥 → 𝑥2𝑐 − ln 𝑥 + 0.5 = 0 (2 points)
2𝑐 −1 2𝑐 −1

→ 𝑥2𝑐 = 0.7 (2 points)


→ (𝑥1𝑐 , 𝑥2𝑐 ) = (0.5, 0.7)

Second trajectory:
𝜆 = −1 (2 points)
𝑥 −𝜆 0.7+1 0.7+1
𝑥1 (𝑡) + 𝑥2 (𝑡) = 𝑥10 + 𝑥20 + 𝜆 ln [𝑥 20
(𝑡)−𝜆
] = 0.5 + 0.7 − ln [ 𝑥 ] = 1.2 − ln [ 𝑥 ] (2 points)
2 2 +1 2 +1

𝑥′1𝑐 = 0.5 (2 points) , 𝑥′2𝑐 =?


1.7 1.7
0.5 + 𝑥′2𝑐 = 1.2 − ln 𝑥 ′ → 𝑥 ′ 2𝑐 + ln 𝑥′ − 0.7 = 0 (2 points)
2𝑐 +1 2𝑐 +1

→ 𝑥′2𝑐 = −0.47 (2 points)


→ (𝑥′1𝑐 , 𝑥′2𝑐 ) = (0.5, −0.47)
Third trajectory:
𝜆 = 1 (2 points)
𝑥 −𝜆 −0.47−1 −1.47
𝑥1 (𝑡) + 𝑥2 (𝑡) = 𝑥10 + 𝑥20 + 𝜆 ln [𝑥 20
(𝑡)−𝜆
] = 0.5 − 0.47 + ln [ ] = 0.03 + ln [ 𝑥 ] (2
2 𝑥2 −1 2 −1
points)
𝑥′′1𝑐 = 0.5 (2 points) , 𝑥′′2𝑐 =?
−1.47 −1.47
0.5 + 𝑥′′2𝑐 = 0.03 + ln 𝑥 ′′ → 𝑥 ′′ 2𝑐 − ln 𝑥′′ + 0.47 = 0 (2 points)
2𝑐 −1 2𝑐 −1

→ 𝑥′′2𝑐 = 0.36 (2 points)


→ (𝑥′′1𝑐 , 𝑥′′2𝑐 ) = (0.5, 0.36)

Fourth trajectory:
𝜆 = −1 (2 points)
𝑥 −𝜆 0.36+1 1.36
𝑥1 (𝑡) + 𝑥2 (𝑡) = 𝑥10 + 𝑥20 + 𝜆 ln [𝑥 20
(𝑡)−𝜆
] = 0.5 + 0.36 − ln [ 𝑥 ] = 0.86 − ln [𝑥 ] (2
2 2 +1 2 +1
points)
𝑥′′′1𝑐 = 0.5 (2 points) , 𝑥′′2𝑐 =?
1.36 1.36
0.5 + 𝑥′′′2𝑐 = 0.86 − ln 𝑥 ′′′ → 𝑥 ′′ 2𝑐 + ln 𝑥′′′ − 0.36 = 0 (2 points)
2𝑐 +1 2𝑐 +1

→ 𝑥′′2𝑐 = −0.29 (2 points)


→ (𝑥′′1𝑐 , 𝑥′′2𝑐 ) = (0.5, −0.29)

(4 points)
The origin is a stable focus. (4 points)
Problem 3 : (20 points)

𝑥̇ 1 = 𝑥2 − 𝑥1 (𝑥12 + 𝑥22 )
𝑥̇ 2 = −𝑥1 − 𝑥2 (𝑥12 + 𝑥22 )
1
𝑉(𝑥) = (𝑥12 + 𝑥22 )
2
𝑉̇ (𝑥) = 𝑥1 𝑥̇ 1 + 𝑥2 𝑥̇ 2 = 𝑥1 (𝑥2 − 𝑥1 (𝑥12 + 𝑥22 )) + 𝑥2 (−𝑥1 − 𝑥2 (𝑥12 + 𝑥22 )) = −2(𝑥12 + 𝑥22 ) < 0
(5 points)
𝑉(𝑥) → ∞ 𝑖𝑓 𝑥 → ∞, 𝑉(𝑥) is not bounded (5 points)

𝑉̇ (𝑥) < 0 ∀𝑥 ≠ 0 (5 points)


The system is globally asymptotically stable. (5 points)

You might also like